Difference between revisions of "2006 AMC 8 Problems/Problem 22"

(Solution)
(Solution)
(2 intermediate revisions by the same user not shown)
Line 31: Line 31:
 
smallest sum, place <math>1</math> in the center cell and <math>2</math> and <math>3</math> in the outer ones. The top
 
smallest sum, place <math>1</math> in the center cell and <math>2</math> and <math>3</math> in the outer ones. The top
 
number will be <math>7</math>. For the largest sum, place <math>9</math> in the center cell and <math>7</math> and <math>8</math> in
 
number will be <math>7</math>. For the largest sum, place <math>9</math> in the center cell and <math>7</math> and <math>8</math> in
the outer ones. This top number will be <math>33</math>. The difference is <math>33 - 7 = \boxed{\textbf{(D)}\  
+
the outer ones. This top number will be <math>33</math>. The difference is <math>33 - 7 = \boxed{\textbf{(D)}\ 26
 
}</math>.
 
}</math>.
  

Revision as of 15:57, 19 October 2020

Problem

Three different one-digit positive integers are placed in the bottom row of cells. Numbers in adjacent cells are added and the sum is placed in the cell above them. In the second row, continue the same process to obtain a number in the top cell. What is the difference between the largest and smallest numbers possible in the top cell?

[asy] path cell=((0,0)--(1,0)--(1,1)--(0,1)--cycle); path sw=((0,0)--(1,sqrt(3))); path se=((5,0)--(4,sqrt(3))); draw(cell, linewidth(1)); draw(shift(2,0)*cell, linewidth(1)); draw(shift(4,0)*cell, linewidth(1)); draw(shift(1,3)*cell, linewidth(1)); draw(shift(3,3)*cell, linewidth(1)); draw(shift(2,6)*cell, linewidth(1)); draw(shift(0.45,1.125)*sw, EndArrow); draw(shift(2.45,1.125)*sw, EndArrow); draw(shift(1.45,4.125)*sw, EndArrow); draw(shift(-0.45,1.125)*se, EndArrow); draw(shift(-2.45,1.125)*se, EndArrow); draw(shift(-1.45,4.125)*se, EndArrow); label("$+$", (1.5,1.5)); label("$+$", (3.5,1.5)); label("$+$", (2.5,4.5));[/asy]

$\textbf{(A)}\ 16\qquad\textbf{(B)}\ 24\qquad\textbf{(C)}\ 25\qquad\textbf{(D)}\ 26\qquad\textbf{(E)}\ 35$

Solution

If the lower cells contain $A, B$ and $C$, then the second row will contain $A + B$ and $B + C$, and the top cell will contain $A + 2B + C$. To obtain the smallest sum, place $1$ in the center cell and $2$ and $3$ in the outer ones. The top number will be $7$. For the largest sum, place $9$ in the center cell and $7$ and $8$ in the outer ones. This top number will be $33$. The difference is $33 - 7 = \boxed{\textbf{(D)}\ 26 }$.

See Also

2006 AMC 8 (ProblemsAnswer KeyResources)
Preceded by
Problem 21
Followed by
Problem 23
1 2 3 4 5 6 7 8 9 10 11 12 13 14 15 16 17 18 19 20 21 22 23 24 25
All AJHSME/AMC 8 Problems and Solutions

The problems on this page are copyrighted by the Mathematical Association of America's American Mathematics Competitions. AMC logo.png